An inequality of matrix normIs there a condition for the following consequence?Orthogonal Inner Product...

Python if-else code style for reduced code for rounding floats

Co-worker team leader wants to inject his friend's awful software into our development. What should I say to our common boss?

What options are left, if Britain cannot decide?

Sailing the cryptic seas

How can you use ICE tables to solve multiple coupled equilibria?

How to make healing in an exploration game interesting

What exactly is this small puffer fish doing and how did it manage to accomplish such a feat?

How to deal with taxi scam when on vacation?

How to change two letters closest to a string and one letter immediately after a string using notepad++

Are there verbs that are neither telic, or atelic?

Is it possible to upcast ritual spells?

how to draw discrete time diagram in tikz

AG Cluster db upgrade by vendor

Why do passenger jet manufacturers design their planes with stall prevention systems?

How to explain that I do not want to visit a country due to personal safety concern?

How to create the Curved texte?

Use void Apex method in Lightning Web Component

Do I need life insurance if I can cover my own funeral costs?

An inequality of matrix norm

Employee lack of ownership

How do I hide Chekhov's Gun?

Is there a data structure that only stores hash codes and not the actual objects?

A link redirect to http instead of https: how critical is it?

What are substitutions for coconut in curry?



An inequality of matrix norm


Is there a condition for the following consequence?Orthogonal Inner Product Proofprove change of basis matrix is unitarya matrix metricMatrix of non-degenerate product invertible?Prove that there is a $uin V$, such that $<u,v_i>$ is greater than zero, for every $i in {{1,..,m}}$.Inner product of dual basisColumn Spaces and SubsetsProve matrix inequality in inner product spaceInduced inner product on tensor powers.













3












$begingroup$


Let $V,W$ be complex inner product spaces. Suppose $T: V to W$ is a linear map, then we define
$$|T|:=sup{|Tv|_{W}:|v|_{V}=1}$$ where $|v_{V}|:=sqrt{langle v,vrangle}$ and $|Tv|_{W}:=sqrt{langle Tv,Tvrangle}$.



Question: Suppose $U_1,ldots,U_k$ and $V_1,ldots,V_k$ are $n {times} n$ unitary matrices. Show that
$$|U_1cdots U_k-V_1cdots V_k| leq sum_{i=1}^{k}|U_i-V_i|$$



I have tried to use triangle inequality for norms and induction but failed. Can anyone give some hints? Thank you!










share|cite|improve this question











$endgroup$

















    3












    $begingroup$


    Let $V,W$ be complex inner product spaces. Suppose $T: V to W$ is a linear map, then we define
    $$|T|:=sup{|Tv|_{W}:|v|_{V}=1}$$ where $|v_{V}|:=sqrt{langle v,vrangle}$ and $|Tv|_{W}:=sqrt{langle Tv,Tvrangle}$.



    Question: Suppose $U_1,ldots,U_k$ and $V_1,ldots,V_k$ are $n {times} n$ unitary matrices. Show that
    $$|U_1cdots U_k-V_1cdots V_k| leq sum_{i=1}^{k}|U_i-V_i|$$



    I have tried to use triangle inequality for norms and induction but failed. Can anyone give some hints? Thank you!










    share|cite|improve this question











    $endgroup$















      3












      3








      3


      1



      $begingroup$


      Let $V,W$ be complex inner product spaces. Suppose $T: V to W$ is a linear map, then we define
      $$|T|:=sup{|Tv|_{W}:|v|_{V}=1}$$ where $|v_{V}|:=sqrt{langle v,vrangle}$ and $|Tv|_{W}:=sqrt{langle Tv,Tvrangle}$.



      Question: Suppose $U_1,ldots,U_k$ and $V_1,ldots,V_k$ are $n {times} n$ unitary matrices. Show that
      $$|U_1cdots U_k-V_1cdots V_k| leq sum_{i=1}^{k}|U_i-V_i|$$



      I have tried to use triangle inequality for norms and induction but failed. Can anyone give some hints? Thank you!










      share|cite|improve this question











      $endgroup$




      Let $V,W$ be complex inner product spaces. Suppose $T: V to W$ is a linear map, then we define
      $$|T|:=sup{|Tv|_{W}:|v|_{V}=1}$$ where $|v_{V}|:=sqrt{langle v,vrangle}$ and $|Tv|_{W}:=sqrt{langle Tv,Tvrangle}$.



      Question: Suppose $U_1,ldots,U_k$ and $V_1,ldots,V_k$ are $n {times} n$ unitary matrices. Show that
      $$|U_1cdots U_k-V_1cdots V_k| leq sum_{i=1}^{k}|U_i-V_i|$$



      I have tried to use triangle inequality for norms and induction but failed. Can anyone give some hints? Thank you!







      linear-algebra matrices functional-analysis norm






      share|cite|improve this question















      share|cite|improve this question













      share|cite|improve this question




      share|cite|improve this question








      edited 1 hour ago









      Bernard

      123k741116




      123k741116










      asked 1 hour ago









      bbwbbw

      51739




      51739






















          1 Answer
          1






          active

          oldest

          votes


















          6












          $begingroup$

          For $k = 1$, the inequality becomes an identity. So, start with the special case $k = 2$:
          $$
          begin{array}{ll}
          & ||U_1 U_2 - V_1 V_2||\ \
          = & ||U_1 U_2 - V_1 U_2 + V_1 U_2 - V_1 V_2||\ \
          = &
          || ( U_1 - V_1) U_2 + V_1 (U_2 - V_2) || \ \
          leq & ||( U_1 - V_1) U_2 || + ||V_1 (U_2 - V_2) ||.\
          end{array}
          $$

          (The last inequality is the triangle inequality.) Now, since $U_2$ is unitary, we have $||U_{2}|| = 1$, so
          $$
          || ( U_1 - V_1) U_2 || leq || U_1 - V_1 ||.
          $$

          A similar bound obtains for $||V_1 (U_2 - V_2) ||$.



          This should give you enough "building blocks".:)






          share|cite|improve this answer











          $endgroup$













          • $begingroup$
            Thank you so much!
            $endgroup$
            – bbw
            1 hour ago






          • 1




            $begingroup$
            You are welcome. I edited to remove the (unjustified) assumption that the matrices commute.:)
            $endgroup$
            – avs
            52 mins ago











          Your Answer





          StackExchange.ifUsing("editor", function () {
          return StackExchange.using("mathjaxEditing", function () {
          StackExchange.MarkdownEditor.creationCallbacks.add(function (editor, postfix) {
          StackExchange.mathjaxEditing.prepareWmdForMathJax(editor, postfix, [["$", "$"], ["\\(","\\)"]]);
          });
          });
          }, "mathjax-editing");

          StackExchange.ready(function() {
          var channelOptions = {
          tags: "".split(" "),
          id: "69"
          };
          initTagRenderer("".split(" "), "".split(" "), channelOptions);

          StackExchange.using("externalEditor", function() {
          // Have to fire editor after snippets, if snippets enabled
          if (StackExchange.settings.snippets.snippetsEnabled) {
          StackExchange.using("snippets", function() {
          createEditor();
          });
          }
          else {
          createEditor();
          }
          });

          function createEditor() {
          StackExchange.prepareEditor({
          heartbeatType: 'answer',
          autoActivateHeartbeat: false,
          convertImagesToLinks: true,
          noModals: true,
          showLowRepImageUploadWarning: true,
          reputationToPostImages: 10,
          bindNavPrevention: true,
          postfix: "",
          imageUploader: {
          brandingHtml: "Powered by u003ca class="icon-imgur-white" href="https://imgur.com/"u003eu003c/au003e",
          contentPolicyHtml: "User contributions licensed under u003ca href="https://creativecommons.org/licenses/by-sa/3.0/"u003ecc by-sa 3.0 with attribution requiredu003c/au003e u003ca href="https://stackoverflow.com/legal/content-policy"u003e(content policy)u003c/au003e",
          allowUrls: true
          },
          noCode: true, onDemand: true,
          discardSelector: ".discard-answer"
          ,immediatelyShowMarkdownHelp:true
          });


          }
          });














          draft saved

          draft discarded


















          StackExchange.ready(
          function () {
          StackExchange.openid.initPostLogin('.new-post-login', 'https%3a%2f%2fmath.stackexchange.com%2fquestions%2f3149889%2fan-inequality-of-matrix-norm%23new-answer', 'question_page');
          }
          );

          Post as a guest















          Required, but never shown

























          1 Answer
          1






          active

          oldest

          votes








          1 Answer
          1






          active

          oldest

          votes









          active

          oldest

          votes






          active

          oldest

          votes









          6












          $begingroup$

          For $k = 1$, the inequality becomes an identity. So, start with the special case $k = 2$:
          $$
          begin{array}{ll}
          & ||U_1 U_2 - V_1 V_2||\ \
          = & ||U_1 U_2 - V_1 U_2 + V_1 U_2 - V_1 V_2||\ \
          = &
          || ( U_1 - V_1) U_2 + V_1 (U_2 - V_2) || \ \
          leq & ||( U_1 - V_1) U_2 || + ||V_1 (U_2 - V_2) ||.\
          end{array}
          $$

          (The last inequality is the triangle inequality.) Now, since $U_2$ is unitary, we have $||U_{2}|| = 1$, so
          $$
          || ( U_1 - V_1) U_2 || leq || U_1 - V_1 ||.
          $$

          A similar bound obtains for $||V_1 (U_2 - V_2) ||$.



          This should give you enough "building blocks".:)






          share|cite|improve this answer











          $endgroup$













          • $begingroup$
            Thank you so much!
            $endgroup$
            – bbw
            1 hour ago






          • 1




            $begingroup$
            You are welcome. I edited to remove the (unjustified) assumption that the matrices commute.:)
            $endgroup$
            – avs
            52 mins ago
















          6












          $begingroup$

          For $k = 1$, the inequality becomes an identity. So, start with the special case $k = 2$:
          $$
          begin{array}{ll}
          & ||U_1 U_2 - V_1 V_2||\ \
          = & ||U_1 U_2 - V_1 U_2 + V_1 U_2 - V_1 V_2||\ \
          = &
          || ( U_1 - V_1) U_2 + V_1 (U_2 - V_2) || \ \
          leq & ||( U_1 - V_1) U_2 || + ||V_1 (U_2 - V_2) ||.\
          end{array}
          $$

          (The last inequality is the triangle inequality.) Now, since $U_2$ is unitary, we have $||U_{2}|| = 1$, so
          $$
          || ( U_1 - V_1) U_2 || leq || U_1 - V_1 ||.
          $$

          A similar bound obtains for $||V_1 (U_2 - V_2) ||$.



          This should give you enough "building blocks".:)






          share|cite|improve this answer











          $endgroup$













          • $begingroup$
            Thank you so much!
            $endgroup$
            – bbw
            1 hour ago






          • 1




            $begingroup$
            You are welcome. I edited to remove the (unjustified) assumption that the matrices commute.:)
            $endgroup$
            – avs
            52 mins ago














          6












          6








          6





          $begingroup$

          For $k = 1$, the inequality becomes an identity. So, start with the special case $k = 2$:
          $$
          begin{array}{ll}
          & ||U_1 U_2 - V_1 V_2||\ \
          = & ||U_1 U_2 - V_1 U_2 + V_1 U_2 - V_1 V_2||\ \
          = &
          || ( U_1 - V_1) U_2 + V_1 (U_2 - V_2) || \ \
          leq & ||( U_1 - V_1) U_2 || + ||V_1 (U_2 - V_2) ||.\
          end{array}
          $$

          (The last inequality is the triangle inequality.) Now, since $U_2$ is unitary, we have $||U_{2}|| = 1$, so
          $$
          || ( U_1 - V_1) U_2 || leq || U_1 - V_1 ||.
          $$

          A similar bound obtains for $||V_1 (U_2 - V_2) ||$.



          This should give you enough "building blocks".:)






          share|cite|improve this answer











          $endgroup$



          For $k = 1$, the inequality becomes an identity. So, start with the special case $k = 2$:
          $$
          begin{array}{ll}
          & ||U_1 U_2 - V_1 V_2||\ \
          = & ||U_1 U_2 - V_1 U_2 + V_1 U_2 - V_1 V_2||\ \
          = &
          || ( U_1 - V_1) U_2 + V_1 (U_2 - V_2) || \ \
          leq & ||( U_1 - V_1) U_2 || + ||V_1 (U_2 - V_2) ||.\
          end{array}
          $$

          (The last inequality is the triangle inequality.) Now, since $U_2$ is unitary, we have $||U_{2}|| = 1$, so
          $$
          || ( U_1 - V_1) U_2 || leq || U_1 - V_1 ||.
          $$

          A similar bound obtains for $||V_1 (U_2 - V_2) ||$.



          This should give you enough "building blocks".:)







          share|cite|improve this answer














          share|cite|improve this answer



          share|cite|improve this answer








          edited 52 mins ago

























          answered 1 hour ago









          avsavs

          3,424513




          3,424513












          • $begingroup$
            Thank you so much!
            $endgroup$
            – bbw
            1 hour ago






          • 1




            $begingroup$
            You are welcome. I edited to remove the (unjustified) assumption that the matrices commute.:)
            $endgroup$
            – avs
            52 mins ago


















          • $begingroup$
            Thank you so much!
            $endgroup$
            – bbw
            1 hour ago






          • 1




            $begingroup$
            You are welcome. I edited to remove the (unjustified) assumption that the matrices commute.:)
            $endgroup$
            – avs
            52 mins ago
















          $begingroup$
          Thank you so much!
          $endgroup$
          – bbw
          1 hour ago




          $begingroup$
          Thank you so much!
          $endgroup$
          – bbw
          1 hour ago




          1




          1




          $begingroup$
          You are welcome. I edited to remove the (unjustified) assumption that the matrices commute.:)
          $endgroup$
          – avs
          52 mins ago




          $begingroup$
          You are welcome. I edited to remove the (unjustified) assumption that the matrices commute.:)
          $endgroup$
          – avs
          52 mins ago


















          draft saved

          draft discarded




















































          Thanks for contributing an answer to Mathematics Stack Exchange!


          • Please be sure to answer the question. Provide details and share your research!

          But avoid



          • Asking for help, clarification, or responding to other answers.

          • Making statements based on opinion; back them up with references or personal experience.


          Use MathJax to format equations. MathJax reference.


          To learn more, see our tips on writing great answers.




          draft saved


          draft discarded














          StackExchange.ready(
          function () {
          StackExchange.openid.initPostLogin('.new-post-login', 'https%3a%2f%2fmath.stackexchange.com%2fquestions%2f3149889%2fan-inequality-of-matrix-norm%23new-answer', 'question_page');
          }
          );

          Post as a guest















          Required, but never shown





















































          Required, but never shown














          Required, but never shown












          Required, but never shown







          Required, but never shown

































          Required, but never shown














          Required, but never shown












          Required, but never shown







          Required, but never shown







          Popular posts from this blog

          Paper upload error, “Upload failed: The top margin is 0.715 in on page 3, which is below the required...

          Emraan Hashmi Filmografia | Linki zewnętrzne | Menu nawigacyjneGulshan GroverGulshan...

          How can I write this formula?newline and italics added with leqWhy does widehat behave differently if I...